You are on page 1of 34

221

Number Series
Q1. In the following number series one of the numbers is wrong. Find out the
wrong one, put it in place of (X) and form a new series based on the same pattern.
Series I: 8, 25, 63, 257, 1281, 7689
Series II: X, Y, Z, P, Q, R
Which of the following statement/s is/are true?
I. R is divisible by 73
II. [P-Q] is a prime number
III. LCM & HCF of Y & Z is 3135 and 3 respectively
A. Only I
B. Both I & II
C. Both II & III
D. Both I & III
E. All I, II & III

Q2. Three series are given. All series are following a different pattern. Read the
given information and answer the related question.
Series I: 61, 82, 105, A, 157, 186
Series II: 23, 47, 96, 195, 394, B
Series III: 35, 36, 39, C, 51, 60
Which of the following is/are true?
i. LCM of A and C is 2860
ii. HCF of A+C and B is 2
iii. B-C is divisible by 7
A. Only I
B. Both I & II
C. Both II & III
D. Both I & III
E. All I, II & III

Q3. Following question given below is based on a missing series pattern and
following that pattern find the relation between A, B and C.
43, 210, 836, _A_, 5008, 5007
210, _B_, 4176, 12525, 25048, 25047
145, 720, _C_, 8625, 17248, 17247
A. A>B<C
B. A>B>C
C. A=B<C
D. A<B=C
E. A=B=C

Q4. Following question given below is based on a missing series pattern and
following that pattern find the relation between A, B and C.
170, _A_, 460, 685, 974
_B_, 412, 581, 806, 1095
145, 266, _C_, 660, 949
A. A<B<C
B. A<B>C
C. A>B=C
D. A>B>C
E. A=B<C
.
Q5. Following question given below is based on a missing series pattern and
following that pattern find the relation between A, B and C.
95, 100, _A_, 127, 153
100, 85, _B_, 80, 110
_C_, 55, 82.5, 206.25, 721.875
A. A=B=C
B. A>B>C
C. A>B=C
D. A=B>C
E. A=C>B
Answer Key:
1. d
2. d
3. a
4. e
5. e

Solution:
Q1.
(8*2)+7 =23 - X
(25*3)-6 = 63
(63*4)+5=257
(257*5)-4=1281
(1281*6)+3=7689
So new series – [follow same pattern as above]
23, 57, 165, 665, 3321, 19929
X Y Z P Q R
So checking all options
i. R= 19929/73=273 so true
ii. P-Q = 3321-665 = 2656 so false
LCM of Y & Z = 57 & 165 =3135 & HCF is 3 so true

Q2.
Series I
61+21 = 82
82+23=105
105+25 = 130 - A
130+27 = 157
157+29 = 186
Series II
23*2 +1 = 47
47*2+2=96
96*2+3=195
195*2+4=394
394*2+5= 793 – B
Series III
35+1=36
36+3=39
39+5=44 - C
44+7=51
51+9=60
So checking all options
i. LCM of A & C = 2860 so true
ii. HCF of 174 & 793 = 2 is not possible as 793 is an odd number
iii. 793-44 = 749 which is divisible by 7

Q3.
43 × 5 – 5 = 210
210 × 4 – 4 = 836
836 × 3 – 3 = 2505
2505 × 2 – 2 = 5008
5008 × 1 – 1 = 5007
A = 2505, B = 1045, C = 2876 [since all follow same pattern]
A>B<C

Q4.
170 + 112 = 291
291 + 132 = 460
460 + 152 = 685
685 + 172 = 974
A = 291, B = 291, C = 435
A=B<C

Q5.
100 – 95 = 5
110 – 100 = 10 (5 + 5)
127 – 110 = 17 (10 + 7)
153 – 127 = 26 (17 + 9)
A = 110
100 – 15 = 85
85 + 20 = 105
105 – 25 = 80
80 + 30 = 110
B = 105
110 × 0.5 = 55
55 × 1.5 = 82.5
82.5 × 2.5 = 206.25
206.25 × 3.5 = 721.875
C = 110
A=C>B
222

Data Interpretation
Directions: Read the data given in the table and graph carefully and answer the
following questions.
Certain number of people works in retail, online and door to door stores. There
are only three type stores and each people works in one or more store. 18/25 of
people were in retail store and people working in only door to door store was
1/36th of people working in retail store. Number of people working in both door
to door store and online store but not in retail store are 110. People working in
only online store are 130 more than the people working in only door to door
store. Number of people working in only retail store is 8/15 more than number of
people working in only online store.
निर्दे श: तानिका और ग्राफ़ में नर्दए गए डे टा को ध्यािपूर्वक पढें और निम्ननिखित प्रश्ोों के
उत्तर र्दें । ररटे ि, ऑििाइि और डोर टू डोर स्टोर्व में निनित र्ोंख्या में िोग काम करते हैं ।
केर्ि तीि प्रकार के स्टोर हैं और प्रत्येक व्यखि एक या अनिक स्टोर में काम करता है ।
18/25 िोग ररटे ि स्टोर में थे और केर्ि डोर टू डोर स्टोर में काम करिे र्ािे िोग ररटे ि
स्टोर में काम करिे र्ािे िोगोों का 1/36र्ाों नहस्सा थे। डोर टू डोर स्टोर और ऑििाइि
स्टोर र्दोिोों में काम करिे र्ािे िेनकि िुर्दरा स्टोर में िहीों काम करिे र्ािे िोगोों की र्ोंख्या
110 है । केर्ि ऑििाइि स्टोर में काम करिे र्ािे िोग केर्ि डोर टू डोर स्टोर में काम
करिे र्ािे िोगोों र्े 130 अनिक हैं । केर्ि ररटे ि स्टोर में काम करिे र्ािे िोगोों की र्ोंख्या
केर्ि ऑििाइि स्टोर में काम करिे र्ािे िोगोों की र्ोंख्या र्े 8/15 अनिक है ।
Q1. What is the number of people who works in retail store, but not only in retail
store?
ऐर्े िोगोों की र्ोंख्या नकतिी है जो केर्ि ररटे ि स्टोर में ही िहीों, बखि ररटे ि स्टोर में भी
काम करते हैं ?
A. 430
B. 450
C. 470
D. 490
E. None of these

Q2. What is the total number of people working in stores?


र्दु कािोों में काम करिे र्ािे िोगोों की कुि र्ोंख्या नकतिी है ?
A. 800
B. 1100
C. 1000
D. 900
E. None of these

Q3. What is the number of people working in online store only?


केर्ि ऑििाइि स्टोर में काम करिे र्ािे िोगोों की र्ोंख्या नकतिी है ?
A. 130
B. 150
C. 155
D. 125
E. None of these

Q4. If number of people working in retail and online but not in door to door are
210 then what % of people is there, who work both in retail and door to door
stores?
यनर्द ररटे ि और ऑििाइि, िेनकि डोर टू डोर स्टोर में काम करिे र्ािे िोगोों की र्ोंख्या
210 है , तो ऐर्े नकतिे प्रनतशत िोग हैं , जो ररटे ि और डोर टू डोर स्टोर र्दोिोों में काम करते
हैं ?
A. 28%
B. 26%
C. 27%
D. 29%
E. None of these

Q5. Number of people working in retail or online store is approximately what


percent of people working in only online or only in retail?
ररटे ि या ऑििाइि स्टोर में काम करिे र्ािे िोगोों की र्ोंख्या केर्ि ऑििाइि या केर्ि
ररटे ि में काम करिे र्ािे िोगोों का िगभग नकतिा प्रनतशत है ?
A. 247.89%
B. 257.89%
C. 254.98%
D. 249.98%
E. None of these
Answer Key:
1. D
2. C
3. B
4. A
5. B

Solution:
Total people = 50x
In Retail store = 50x × 18/25 = 36x
In Only Door to Door store = 36x/36 = x
In Only Online store = x + 130
36x + x + 110 + (x + 130) = 50x
38x + 240 = 50x
12x = 240
x = 20
Total people = 50 × 20 = 1000

Q1.
People in Retail store but not only in Retail store = 720 – 230 = 490
Q2.
Total People working in store = 1000
Q3.
People only in Online store = 20 + 130 = 150
Q4.
People in Retail & Door to Door store = 720 – 230 – 210 = 280
% = 280/1000 × 100 = 28%
Q5.
People in Retail or Online store = 720 + 110 + 150 = 980
People in Only Online & Only Retail = 230 + 150 = 380
% = 980/380 × 100 = 257.89%
223

Data Interpretation
Directions: Read the data given in the table and graph carefully and answer the
following questions.
A bank accepts deposits in two forms (RD and FD) and it provides loan. Interest
rate given on RD and FD is 5% and 4% respectively and interest charged on loan
amount is 6%. Interest is calculated every year and rate of interest is given on
yearly basis. How interest amount is calculated: On the sum of Rs. 500 in RD
deposit, interest amount for 1 year = 5% of 500 = Rs. 25 and so on. Bar graph
given below shows: (1) the sum of difference between amounts deposited in RD &
FD and amount of loan given by Bank in 4 different years. (2) Amount deposited in
RD and FD in 4 different years. (3) Difference between interest on FD and interest
on loan (Interest on loan amount > Interest on FD amount).
निर्दे श: तानिका और ग्राफ़ में नर्दए गए डे टा को ध्यािपूर्वक पढें और निम्ननिखित प्रश्ोों के
उत्तर र्दें । एक बैंक र्दो रूपोों (आरडी और एफडी) में जमा स्वीकार करता है और ऋण
प्रर्दाि करता है । आरडी और एफडी पर ब्याज र्दर क्रमशः 5% और 4% है और ऋण रानश
पर ब्याज 6% है । ब्याज की गणिा हर र्ाि की जाती है और ब्याज र्दर र्ानषवक आिार पर
र्दी जाती है । ब्याज रानश की गणिा कैर्े की जाती है : रुपये की रानश पर। आरडी जमा में
500, 1 र्षव के निए ब्याज रानश = 500 का 5% = रु. 25 र्गैरह. िीचे नर्दया गया बार ग्राफ
र्दशाव ता है : (1) 4 अिग-अिग र्षों में आरडी और एफडी में जमा रानश और बैंक द्वारा नर्दए
गए ऋण की रानश के बीच अोंतर का योग। (2) 4 अिग-अिग र्षों में आरडी और एफडी में
जमा की गई रानश। (3) एफडी पर ब्याज और िोि पर ब्याज के बीच अोंतर (िोि रानश पर
ब्याज > एफडी रानश पर ब्याज)।
Q1. If loss of the Bank in any particular year is the amount that it loses after giving
interest amount on deposits and taking interest on Loan, then what is the total
loss of Bank in 2015?
यनर्द नकर्ी नर्शेष र्षव में बैंक का घाटा र्ह रानश है जो र्ह जमा पर ब्याज रानश र्दे िे और
ऋण पर ब्याज िेिे के बार्द िोता है , तो 2015 में बैंक का कुि घाटा नकतिा है ?
A. Rs.20
B. Rs.40
C. Rs.50
D. Rs.15
E. None of these

Q2. What is the ratio of total interest amount given by Bank on both types of
deposited amount to the total interest amount received by Bank on Loans in all
the 4 years together?
र्भी 4 र्षों में बैंक द्वारा र्दोिोों प्रकार की जमा रानश पर र्दी गई कुि ब्याज रानश का बैंक
द्वारा ऋण पर प्राप्त कुि ब्याज रानश र्े अिुपात क्या है ?
A. 50:67
B. 49:66
C. 80:57
D. 35:71
E. None of these
Q3. What is the difference between averages deposited amount on RD in Bank in
all 4 years to the average amount of loan given by Bank in all 4 years together?
र्भी 4 र्षों में बैंक में आरडी पर जमा की गई और्त रानश और र्भी 4 र्षों में बैंक द्वारा
नर्दए गए ऋण की और्त रानश के बीच नकतिा अोंतर है ?
A. 3100
B. 4000
C. 4500
D. 2800
E. None of these

Q4. If in 2019, total profit amount gained by Bank is Rs. 140 and amount
deposited in FD is half of the loan amount given. If total amount deposited in RD
is Rs. 2000, then what is the interest amount given by Bank on total deposits on
FD?
यनर्द 2019 में, बैंक द्वारा प्राप्त कुि िाभ रानश रु. 140 और एफडी में जमा रानश र्दी गई
ऋण रानश का आिा है । यनर्द आरडी में जमा की गई कुि रानश रु. 2000 है , तो FD पर कुि
जमा पर बैंक द्वारा र्दी जािे र्ािी ब्याज रानश क्या है ?
A. 100
B. 120
C. 130
D. 140
E. None of these
Q5. What is the average profit earned by the bank from 2015 to 2018?
2015 र्े 2018 तक बैंक द्वारा अनजवत और्त िाभ क्या है ?
A. 117.5
B. 127.5
C. 147.5
D. 137.5
E. None of these
Answer Key:
1. A
2. B
3. C
4. B
5. B

Solution:
In 2015, RD amount = 2x Rs. , FD amount = x Rs. ,
Loan amount = 13000 – (2x – x) = 13000 – x Rs.
(13000 – x) × 0.06 – x × 0.04 = 380
780 – 0.06x – 0.04x = 380
0.1x = 400
x = 4000 Rs.
Similarly solve for other years.
Year 2015 2016 2017 2018 Total

FD 4000 6000 3000 5000 18000


RD 8000 4000 2000 1000 15000
Loan 9000 8000 9000 7000 33000

Q1.
Loss in 2015
= 8000 × 0.05 + 4000 × 0.04 – 9000 × 0.06
= 400 + 160 – 540 = 20 Rs.
Q2.
Total Interest amount from deposits: Total Interest from Loan
= (18000 × 0.04 + 15000 × 0.05): (33000 × 0.06)
= (720 + 750) : 1980 = 1470 : 1980 = 49 : 66
Q3.
Average Amount of Loan – Average Amount of RD = 33000/4 – 15000/4 =
18000/4 = 4500
Q4.
In 2019, Amount of FD = x Rs. , Amount of Loan = 2x Rs.
2x × 0.06 – [x × 0.04 + 2000 × 0.05] = 140
0.12x – 0.04x – 100 = 140
0.08x = 240
x = 3000
Interest on FD amount = 3000 × 0.04 = 120 Rs.
Q5.
Average Profit from 2015 to 2108 = (33000 × 0.06 – 18000 × 0.04 – 15000 ×
0.05)/4
= (33000 – 720 – 750)/4 = 510/4 = 127.5 Rs.
224

Quantity Based questions (Arithmetic)

In the question, two quantities i.e. Quantity I and Quantity II are given. Solve
the given quantities to establish the correct relation between them and choose
the correct option.

प्रश्न में, दो मात्राएँ अर्ाात् मात्रा I और मात्रा II दी गई हैं । उनके बीच सही संबंध स्र्ापित करने के
पिए दी गई मात्राओं को हि करें और सही पिकल्प चुनें।

Q1. A, B and C started a business with initial investment of Rs. 'x + 1500', Rs. 'x'
and Rs. 'x - 1000'.

Quantity I: If the profit earned after one year is Rs. 17760 and profit share of B is
Rs. 5760, then find the initial investment of A.

Quantity II : If A, B and C invested for 'y' months, '2y' months and '3y' months
respectively, then the profit share of C is Rs. 10000 from total profit of Rs. 23750,
then find the amount invested by B.

A, B और C िे रुपये के प्रारों नभक निर्ेश के र्ाथ एक व्यर्र्ाय शुरू नकया। 'x + 1500',
रु .'x' और रु .'x - 1000'.

मात्रा I: यनर्द एक र्षव के बार्द अनजवत िाभ रु .17760 और बी का िाभ नहस्सा रु .5760 है ,
तो A का प्रारों नभक निर्ेश ज्ञात कीनजए।

मात्रा II: यनर्द A, B और C िे क्रमशः 'y' महीिे, '2y' महीिे और '3y' महीिे के निए निर्ेश
नकया है , तो C का िाभ नहस्सा रु कुि िाभ .10000 रु .23750 है , तो बी द्वारा निर्ेश की
गई रानश ज्ञात कीनजए।
A. QI>QII
B. QI<QII
C. QII≥QI
D. QI≥QII
E. QI=QII

Q2. P, Q and R can cover 120 meters in 'x' seconds, 'x - 2' seconds and 'x + 3'
seconds respectively. The ratio between speeds of P and R is 5: 4.

Quantity I: In a race of 300 meters between Q and R, what will be the distance
between them when winner will finish the race?

Quantity II: In a race of 320 meters between P and R. What will be the distance
between them when winner will finish the race?

P, Q और R क्रमशः 'x' र्ेकोंड, 'x - 2' र्ेकोंड और 'x + 3' र्ेकोंड में 120 मीटर की र्दूरी
तय कर र्कते हैं। P और R की गनत के बीच का अिुपात 5:4 है ।

मात्रा I: Q और R के बीच 300 मीटर की र्दौड़ में, जब नर्जेता र्दौड़ पूरी करे गा तो उिके
बीच की र्दूरी क्या होगी?

मात्रा II: P और R के बीच 320 मीटर की र्दौड़ में। जब नर्जेता र्दौड़ पूरी करे गा तो उिके
बीच की र्दूरी क्या होगी?

A. QI>QII
B. QI<QII
C. QII≥QI
D. QI≥QII
E. QI=QII
Q3.

Quantity I: The digits at unit and hundredth places of a three - digit number 'X' are
interchanged to get another three - digit number 'Y'. If the difference between 'X'
and 'Y' is 198 and the digit at tens place is '8', then how many different values can
'X' take?

Quantity II: A woman takes 15 days to finish a work. A man can do the same work
in 25 days. A child is 50% as efficient as a woman. If 6 children and 5 men take '4X'
days to do the work that 12 women together can finish in 16 days, then find the
value of 'X'.

मात्रा I: तीि अोंकोों की र्ोंख्या 'X' की इकाई और र्ौर्ें स्थाि के अोंकोों को एक और तीि
अोंकोों की र्ोंख्या 'Y' प्राप्त करिे के निए आपर् में बर्दि नर्दया जाता है । यनर्द 'X' और 'Y' के
बीच का अोंतर 198 है और र्दहाई के स्थाि पर अोंक '8' है , तो 'X' नकतिे नभन्न माि िे
र्कता है ?

मात्रा II: एक मनहिा को एक काम पूरा करिे में 15 नर्दि िगते हैं । एक आर्दमी उर्ी कायव
को 25 नर्दिोों में कर र्कता है । एक बच्चा एक मनहिा की तुििा में 50% अनिक कुशि
होता है । यनर्द 6 बच्चे और 5 पुरुष उर् कायव को करिे में '4X' नर्दि िेते हैं नजर्े 12
मनहिाएँ नमिकर 16 नर्दिोों में पूरा कर र्कती हैं , तो 'X' का माि ज्ञात कीनजए।

A. QI>QII
B. QI<QII
C. QII≥QI
D. QI≥QII
E. QI=QII

Q4.

Quantity I: Vivek and Vishal entered into a business investing their money in the
ratio of 17: 12, respectively and the respective ratio of time for which they made
their investment is 12: 19. If profit share of Vishal is Rs. 741, then find the total
profit earned by them together.

Quantity II : A shopkeeper bought a chair and marked it 70% above the cost price
and sold it after giving two successive discounts of 20% and 25% respectively for
Rs. 918. Find the cost price of table whose cost price is 35% more than that of
chair.

मात्रा I: नर्र्ेक और नर्शाि िे क्रमशः 17:12 के अिुपात में अपिा पैर्ा निर्ेश करके एक
व्यर्र्ाय में प्रर्ेश नकया और नजर् र्मय के निए उन्ोोंिे अपिा निर्ेश नकया उर्का
र्ोंबोंनित अिुपात 12:19 है । यनर्द नर्शाि का िाभ नहस्सा रुपये है । 741 है , तो उिके द्वारा
अनजवत कुि िाभ ज्ञात कीनजए।

मात्रा II: एक र्दु कािर्दार िे एक कुर्ी िरीर्दी और उर् पर िागत मूल्य र्े 70% अनिक मूल्य
अोंनकत नकया और उर्े क्रमशः 20% और 25% की र्दो क्रनमक छूट र्दे िे के बार्द रुपये में
बेच नर्दया। 918. उर् मेज का क्रय मूल्य ज्ञात कीनजए नजर्का क्रय मूल्य कुर्ी के क्रय मूल्य
र्े 35% अनिक है ।

A. QI>QII
B. QI<QII
C. QII≥QI
D. QI≥QII
E. QI=QII

Q5.

Quantity I: A box contains 4 Yellow shirts, 5 Pink shirts and 3 Blue shirts. The
probability of picking two shirts of the same colour from the box at random and
without replacement is 'X'. Find the value of '99X + 1.5'.

Quantity II: 7 friends meet at a party and each friend shakes the hand of every
other friend in the party exactly once. Find the number of handshakes that took
place in the party.
मात्रा I: एक बॉक्स में 4 पीिी शटव , 5 गुिाबी शटव और 3 िीिी शटव हैं। नडब्बे र्े यादृखिक
रूप र्े और नबिा प्रनतस्थापि के एक ही रों ग की र्दो शटव चुििे की र्ोंभार्िा 'X' है । '99X +
1.5' का माि ज्ञात कीनजए।

मात्रा II: 7 र्दोस्त एक पाटी में नमिते हैं और प्रत्येक र्दोस्त पाटी में हर र्दूर्रे र्दोस्त र्े ठीक
एक बार हाथ नमिाता है । पाटी में हुए हाथ नमिािे की र्ोंख्या ज्ञात कीनजए।

A. QI>QII
B. QI<QII
C. QII≥QI
D. QI≥QII
E. QI=QII

Answers:

1. A
2. A
3. A
4. A
5. A
Solution:

Q1.
Quantity I
x/(3x + 500) = 5760/17760 = 12/37
37x = 36x + 6000
x = 6000
Initial Investment of A = 6000 + 1500 = 7500 Rs.

Quantity II
{(x - 1000) × 3y}/{(x + 1500) × y + x × 2y + (x - 1000) × 3y} = 10000/23750
3(x - 1000)/(x + 1500 + 2x + 3x - 3000) = 8/19
3(x - 1000)/(6x - 1500) = 8/19
19 × (x - 1000) = 8 × (2x - 500)
19x - 19000 = 16x - 4000
3x = 15000
x = 5000
Amount Invested by B = x = 5000 Rs.
Hence,
Quantity I > Quantity II

Q2.

120 = 5 × x = 4 × (x + 3)
5x = 4x + 12
x = 12
Speed of P = 120/12 = 10 m/s
Speed of Q = 120/(12 - 2) = 120/10 = 12 m/s
Speed of R = 120/(12 + 3) = 120/15 = 8 m/s

Quantity I
Distance between Q & R, when winner finish the race = 300 – [300/12] × 8 = 300 -
200 = 100 m

Quantity II
Distance between P & R, when winner finish the race = 320 – [320/10] × 8 = 320 -
256 = 64 m
Hence,
Quantity I > Quantity II

Q3.

Quantity I
Let X = 100a + 80 + b
Y = 100b + 80 + a
If a > b
X - Y = 198
(100a + 80 + b) - (100b + 80 + a) = 198
99a - 99b = 198
a-b=2
(a, b) = (9, 7), (8, 6), (7, 5), (6, 4), (5, 3), (4, 2), (3, 1)
If a < b
Y - X = 198
(100b + 80 + a) - (100a + 80 + b) = 198
99b - 99a = 198
b-a=2
(b, a) = (9, 7), (8, 6), (7, 5), (6, 4), (5, 3), (4, 2), (3, 1)
Number of Possible values of X = 7 + 7 = 14

Quantity II
Woman - 15 5
} 75 {
Man - 25 3

Efficiency of Child = 5 × 0.5 = 2.5


(6 × 2.5 + 5 × 3) × 4X = 12 × 5 × 16
(15 + 15) × X = 3 × 5 × 16
2X = 16
X=8
Hence,
Quantity I > Quantity II

Q4.

Quantity I
Profit Ratio, Vivek: Vishal = (17 × 12) : (12 × 19) = 17 : 19
Total Profit = 741 × 36/19 = 1404 Rs.

Quantity II
CP of Chair = 918/ (0.75 × 0.8 × 1.7) = 918/1.02 = 900 Rs.
CP of Table = 900 × 1.35 = 1215 Rs.
Hence,
Quantity I > Quantity II

Q5.

Quantity I
X = (4C2 + 5C2 + 3C2)/12C2 = (6 + 10 + 3)/66 = 19/66
99X + 1.5 = 99 × 19/6 + 1.5 = 28.5 + 1.5 = 30

Quantity II
Number of Handshakes = 7C2 = 21
Hence,
Quantity I > Quantity II
225 x

Arithmetic Based Mixed Question Type

Q1. Amit and Ritik invested certain amount of money in two Schemes A and B.
Scheme A offers simple interest at rate of 12.5% for 4 years and Scheme B offers
compound interest at rate of 20% for 2 years. Ratio between the total amount
invested by Amit and Ritik is 3: 4. Amit invested 25% of his amount in Scheme A
and rest in Scheme B. Ritik invested 80% of his money in Scheme A and rest in B. If
Amit invested total Rs.___, then the difference between the total interest earned
by Amit and Ritik from both Schemes is Rs.___. Which of the following options
can fill the blanks?

अनमत और ररनतक िे र्दो योजिा A और B में कुछ ििरानश निर्ेश की। योजिा A 4 र्षों के
निए 12.5% की र्दर र्े र्ािारण ब्याज प्रर्दाि करती है और योजिा B 2 र्षों के निए 20%
की र्दर र्े चक्रर्ृखि ब्याज प्रर्दाि करती है । अनमत और ररनतक द्वारा निर्ेश की गई कुि
रानश के बीच का अिुपात 3: 4 है । अनमत िे अपिी रानश का 25% योजिा A में निर्ेश नकया
और शेष रानश योजिा B में निर्ेश नकया। ररनतक िे अपिे िि का 80% योजिा A में निर्ेश
नकया और शेष रानश B में निर्ेश नकया। यनर्द अनमत िे कुि रानश निर्ेश की रु.___, तो
र्दोिोों योजिाओों र्े अनमत और ररनतक द्वारा अनजवत कुि ब्याज के बीच का अोंतर रु.___ है ।
निम्ननिखित में र्े कौि र्ा नर्कल्प ररि स्थाि को भर र्कता है ?

I. Rs.12500 & Rs.2348


II. Rs.15000 & Rs.2935
III. Rs.18000 & Rs.3522
IV. Rs.24000 & Rs. 4696
V. None of these

A. Only I
B. Both I & II
C. Only I, II& III
D. All of these
E. None of these

Q2. P and R together can complete a work in 12 days. Time taken by Q, R and S
together to complete 74% of the work is equal to time taken by Q, R and T
together to complete 84% of the same work. T is 50% more efficient than S. T
alone can complete the work in 24 days. Q, R, S and T together can complete 78%
of the same work in 5.4 days. From given question, which of the following
statements can be determined?

I. Time taken by P alone to complete the work


II. Time taken by Q and R together to complete the work
III. Time taken by S and P together to complete the work
IV. Time taken by Q alone to complete the work

P और R नमिकर एक कायव को 12 नर्दिोों में पूरा कर र्कते हैं । Q, R और S द्वारा नमिकर


74% काम पूरा करिे में निया गया र्मय, Q, R और T द्वारा नमिकर उर्ी काम का 84%
पूरा करिे में निए गए र्मय के बराबर है। T, S र्े 50% अनिक कुशि है । T अकेिे कायव
को 24 नर्दिोों में पूरा कर र्कता है । Q, R, S और T नमिकर र्माि कायव का 78% भाग 5.4
नर्दिोों में पूरा कर र्कते हैं । नर्दए गए प्रश् र्े , निम्ननिखित में र्े कौि र्ा कथि नििाव ररत नकया
जा र्कता है ?
I. P द्वारा अकेिे कायव पूरा करिे में निया गया र्मय
II. कायव को पूरा करिे में Q और R द्वारा एक र्ाथ निया गया र्मय
III. कायव को पूरा करिे में S और P द्वारा नमिकर निया गया र्मय
IV. कायव को अकेिे Q द्वारा पूरा करिे में निया गया र्मय

A. Only I & II
B. Only II
C. Only II & III
D. Only I, II & IV
E. None of these

Q3. Riya bought three items P, Q and R and she marked them some percentage
above their cost price and sold them after giving some discount. Discount given
on item Q is Rs. 64 more than that of P. She earns a profit of Rs. 66 on selling item
Q. Difference between the cost price of items P and Q is Rs. 150 which is thrice of
the difference between the cost prices of items Q and R. Difference between the
marked price of items P and Q is Rs. 160 which is Rs. 20 more than difference
between the marked prices of items Q and R. She earns a profit of 12.5% and gave
a discount of Rs. 200 on item R. She does not loose money on selling any item.
Marked price of item R is Rs. 1100. From the statement which of the following can
be determined?
ररया िे तीि र्स्तुएँ P, Q और R िरीर्दीों और उन्ोोंिे उि पर उिकी िागत मूल्य र्े कुछ
प्रनतशत ऊपर अोंनकत नकया और कुछ छूट र्दे िे के बार्द उन्ें बेच नर्दया। आइटम Q पर र्दी
गई छूट रु .P र्े 64 रु अनिक है । र्ह रु. का िाभ कमाती है । र्स्तु .Q बेचिे पर 66 रु र्स्तु .
P और Q के िागत मूल्य के बीच अोंतर रु .150 जो र्स्तु Q और R के िागत मूल्य के बीच
अोंतर का तीि गुिा है । र्स्तु P और Q के अोंनकत मूल्य के बीच अोंतर रु है । जो नक .160
रुपये है । र्स्तु Q और R के अोंनकत मूल्योों के बीच अोंतर र्े 20 रुपये अनिक है । र्ह 12.5%
का िाभ कमाती है और रुपये की छूट र्दे ती है । र्स्तु R पर 200 रुर्ह . नकर्ी भी र्स्तु को
बेचिे पर पैर्े िहीों गोंर्ाती। र्स्तु R का अोंनकत मूल्य रु .1100. कथि र्े निम्ननिखित में र्े
क्या नििाव ररत नकया जा र्कता है ?

I. SP of Item Q
II. MP of item P
III. CP of item R
IV. Discount on item P

A. Only I & II
B. Only II & III
C. Only III
D. Only A, C & D
E. None of these

Q4. Quantity 1: A, B and C entered into a business with initial investments of Rs


1620, Rs2430 and Rs1890 respectively. After 8 months, A added Rs 540 more
while B and C each withdrew Rs 810. Find the profit share of B out of total profit
of Rs 13950 at the end of the year.

Quantity 2: Rs 3000 invested at a rate of R% per annum compounded annually


becomes Rs5184 at the end of 3 years. Find the interest earned on Rs2400 in 2
years at R% per annum simple interest.
मात्रा 1: A, B और C िे क्रमशः 1620 रुपये, 2430 रुपये और 1890 रुपये के शुरुआती
निर्ेश के र्ाथ एक व्यर्र्ाय में प्रर्ेश नकया। 8 महीिे के बार्द, A िे 540 रुपये और जोड़े
जबनक B और C प्रत्येक िे 810 रुपये निकािे। र्षव के अोंत में 13950 रुपये के कुि िाभ में
र्े B का िाभ नहस्सा ज्ञात कीनजए।

मात्रा 2: प्रनत र्षव R% की र्दर र्े निर्ेश नकया गया 3000 रुपये, र्ािािा चक्रर्ृखि के र्ाथ 3
र्ाि के अोंत में 5184 रुपये हो जाता है । R% प्रनत र्षव र्ािारण ब्याज की र्दर र्े 2 र्षों में
2400 रुपये पर अनजवत ब्याज ज्ञात कीनजए।
A. Quantity I < Quantity II
B. Quantity I ≥ Quantity II
C. Quantity I > Quantity II
D. Quantity I ≤ Quantity II
E. Quantity I = Quantity II or relation can't be established

Q5. A vendor bought ___ kg of oranges at Rs.30/kg. He later found out that ___ kg
of oranges had gone bad and couldn’t be sold now. So he decided to sell the
remaining oranges at Rs.45/kg. In this transaction he earned a profit of Rs.___.

The value given in each of the following options will fill in the above given blanks
in the same order in which it is given to make the above statement true.

एक नर्क्रेता िे 30 रुपये प्रनत नकिोग्राम की र्दर र्े ___ नकिोग्राम र्ोंतरे िरीर्दे । बार्द में उन्ें
पता चिा नक ___ नकिो र्ोंतरे िराब हो गए हैं और अब बेचे िहीों जा र्केंगे। इर्निए
उन्ोोंिे बचे हुए र्ोंतरे को 45 रुपये प्रनत नकिोग्राम के नहर्ाब र्े बेचिे का फैर्िा नकया।
इर् िेि र्दे ि में उर्िे-___ रुपये का िाभ कमाया। निम्ननिखित नर्कल्पोों में र्े प्रत्येक में
नर्दया गया माि ऊपर नर्दए गए ररि स्थाि को उर्ी क्रम में भर र्दे गा नजर्में उपरोि कथि
को र्त्य बिािे के निए नर्दया गया है ।

a. 29,9, 15
b. 77, 25, 30
c. 45, 13, 90

A. Only b
B. Only a and b
C. Only a and c
D. Only c and b
E. Only c

Q6.
Quantity I: Ratio of present ages of P and Q is 6:7 respectively while the ratio of
present ages of R and S is 8:9. Ratio of the age of Q after two years to age of R
seven years ago is 6:5. Sum of the present ages of P, Q, R and S is 120, then find
average of the ages of P and S.
Quantity II: If the average of ‘q’ numbers is 20. If half of the numbers are
decreased by 20 and half of the others are increased by 16, then what is the new
average?

मात्रा I: P और Q की र्तवमाि आयु का अिुपात क्रमशः 6:7 है जबनक R और S की र्तवमाि


आयु का अिुपात 8:9 है । र्दो र्षव बार्द Q की आयु और र्ात र्षव पहिे R की आयु का
अिुपात 6:5 है । P, Q, R और S की र्तवमाि आयु का योग 120 है , तो P और S की आयु का
और्त ज्ञात कीनजए।

मात्रा II: यनर्द 'q' र्ोंख्याओों का और्त 20 है । यनर्द आिी र्ोंख्याओों में 20 की कमी की जाती
है और अन्य की आिी र्ोंख्याओों में 16 की र्ृखि की जाती है , तो िया और्त क्या है ?

A. Quantity I > Quantity II


B. Quantity I ≥ Quantity II
C. Quantity I < Quantity II
D. Quantity I ≤ Quantity II
E. Quantity I = Quantity II or relation can't be established
Answer Key

1. D
2. B
3. C
4. C
5. D
6. A

Solution

Q1.
Total Amount of Amit = x Rs.
Total Amount of Ritik = 4x/3 Rs.
Interest received by Amit = x × {0.25 × 0.125 × 4 + 0.75 × (1.22 - 1)}
= x × (0.125 + 0.33) = 0.455x Rs.
Interest received by Ritik = 4x/3 × (0.8 × 0.125 × 4 + 0.2 × (1.22 - 1)}
= 4x/3 × (0.4 + 0.088) = 4x/3 × 0.488 = 1.952x/3 Rs.
Difference = 1.952x/3 - 0.455x = 0.587x/3 Rs.
Option A, x = 12000 Rs.
Difference = 0.587/3 × 12000 = 2348 Rs. (Right)
Option B, x = 15000 Rs.
Difference = 0.587/3 × 15000 = 2935 Rs. (Right)
Option C, x = 18000 Rs.
Difference = 0.587/3 × 18000 = 3522 Rs. (Right)
Option D, x = 24000 Rs.
Difference = 0.587/3 × 24000 = 4696 Rs. (Right)
All of these

Q2.
Time taken by T alone = 24 days
Time taken by S alone = 24 × 1.5 = 36 days
P & R - 12 6
T- 24 } 72 { 3
S- 36 2
Efficiency of Q & R = (72 × 0.78)/5.4 - (3 + 2) = 10.4 - 5 = 5.4
Individual efficiency of P, Q & R cannot be determined hence
Time taken by Q & R together to complete the work = 72/5.4 = 13.33 days
Only B

Q3.
MP of Item R = 1100 Rs.
SP of Item R = 1100 - 200 = 900 Rs.
CP of Item R = 900/1.125 = 800 Rs. (C)
Since there is no clear description to calculate MP, SP & CP of Item Q & Item P, no
other value cannot be determined.
Only C

Q4.
Quantity 1: Ratio of profit of A, B and C= (1620*8+2160*4) : (2430*8+1620*4) :
(1890*8+1080*4)=10:12:9
31units–Rs13950
1unit–Rs450
12units–Rs12*450=Rs5400
Profit share of B=Rs5400

Quantity 2: Ratio of principal to amount=5:6


Rate of interest(R) = 20%
SI= 40% of 2400 = Rs960

Q1>Q2
Q5.
From a,
Profit = (29-9)*45 – 29*30 = 900-870 = 30 (not equal to 15)
So a doesn’t fit in the blanks
From b,
Profit = (77-25)*45 – 77*30 = 2340-2310 = 30
So b fits in the blanks
From c,
Profit = (45-13)*45 – 45*30 = 1440 – 1350 = 90
So c fits in the blanks.

Q6.
Let the age of P = 6x years
Therefore, age of Q = 7x years
Age of R=8 years
Therefore, age of S = 36years
(7x+2)/(8y–7) = 6/5… Equation(1)
Sum of ages =120
13x+17y=120….. Equation(2)
Solving Equation(1) & Equation(2)
x=4
y=4
P=24years
S=36years
Average of P and S=24+36/2=30years (quantity 1)

Quantity 2 :

Sum of q numbers = 20q

So new sum = 20q + (0.5q*16) – (0.5q*20) = 18q


New average = 18q/q = 18

Q1>Q2

You might also like